What is the expression for the inverse matrix of A?

Click For Summary
SUMMARY

The inverse matrix of A, represented as A^-1, can be derived from the equation A^2 = (a+d)A - (ad-bc)I, where A is a 2x2 matrix [a b; c d] and I is the identity matrix. Given that ad-bc is not equal to zero, the expression for A^-1 is A^-1 = 1/(ad-bc)((a+d)I - A). This formula confirms that the determinant (ad-bc) is crucial for the existence of the inverse matrix. The discussion highlights the relationship between matrix multiplication and the derivation of the inverse.

PREREQUISITES
  • Understanding of 2x2 matrix operations
  • Knowledge of matrix determinants, specifically ad-bc
  • Familiarity with the identity matrix
  • Basic algebraic manipulation skills
NEXT STEPS
  • Study the derivation of the inverse matrix for larger matrices using Gaussian elimination
  • Learn about the properties of determinants in matrix theory
  • Explore the application of matrix inverses in solving linear equations
  • Investigate the role of eigenvalues and eigenvectors in matrix analysis
USEFUL FOR

Students and professionals in mathematics, particularly those studying linear algebra, as well as anyone involved in computational mathematics or engineering requiring matrix manipulations.

Briane92
Messages
4
Reaction score
0
1. a) Prove the following holds for A
A is a matrix [a b, c d]

I is identity matrix.

A^2 = (a+d)A-(ad-bc)I.

b) Assuming ad-bc not equal to 0, use a) to obtain an expression for A^-1.

The Attempt at a Solution


I proved the first equation, but I'm not seeing where it relates to the inverse. I know that ad-bc is the determinate. At first I was going to write A^-1 in terms of a,d,b,c in a matrix but I realize that this was done in class and its asking for an equation similar to the first one.

I just want a couple of hints, because I'm stuck.
 
Physics news on Phys.org
Well, if
\begin{bmatrix}w & x \\ y & z\end{bmatrix}
is inverse to
\begin{bmatrix}a & b \\ c & d \end{bmatrix}
then we must have
\begin{bmatrix}w & x \\ y & z\end{bmatrix}\begin{bmatrix}a & b \\ c & d \end{bmatrix}= \begin{bmatrix}aw+ cx & bw+ cd \\ ay+ cz & by+ cz \end{bmatrix}= \begin{bmatrix}1 & 0 \\ 0 & 1\end{bmatrix}

That gives you four equations to solve for w, x, y, and z.
 
Thanks for the reply.
We did that in class, I have it in my notes. I think the question is asking for something along this lines of
A^-1 = (b+a)A-(bc+da)I.
That isn't right as I just made it up, but that's the type of equation I think I suppose to come up with from this A^2 = (a+d)A-(ad-bc)I equation. I did the work to show that is true. But I don't see the relation to the inverse except (ad-bc), the determinate, determines if A is invertible.
 
Briane92 said:
1. a) Prove the following holds for A
A is a matrix [a b, c d]

I is identity matrix.

A^2 = (a+d)A-(ad-bc)I.

b) Assuming ad-bc not equal to 0, use a) to obtain an expression for A^-1.

The Attempt at a Solution


I proved the first equation, but I'm not seeing where it relates to the inverse. I know that ad-bc is the determinate. At first I was going to write A^-1 in terms of a,d,b,c in a matrix but I realize that this was done in class and its asking for an equation similar to the first one.

I just want a couple of hints, because I'm stuck.

If B = A^(-1) exists, what do you get if you multiply your equation for A^2 by B on both sides?

RGV
 
(A^-1) A^2 = (A^-1) ((a+d)A-(ad-bc)I)
(A^-1)(A)(A)= " " ""
IA = " " ""
A= (a+d)I-(ad-bc)A^-1

edit
A^-1 = 1/(ad-bc)(a+d)I- 1/(ad-bc)A

just check with calculator and it works.

Thanks Ray and Ivy.
 
Last edited:
Briane92 said:
(A^-1) A^2 = (A^-1) ((a+d)A-(ad-bc)I)
(A^-1)(A)(A)= " " ""
IA = " " ""
A= (a+d)I-(ad-bc)A^-1

Ok, so far. Now just solve that equation for A^(-1).
 
Question: A clock's minute hand has length 4 and its hour hand has length 3. What is the distance between the tips at the moment when it is increasing most rapidly?(Putnam Exam Question) Answer: Making assumption that both the hands moves at constant angular velocities, the answer is ## \sqrt{7} .## But don't you think this assumption is somewhat doubtful and wrong?

Similar threads

Replies
2
Views
2K
Replies
4
Views
2K
  • · Replies 5 ·
Replies
5
Views
2K
  • · Replies 3 ·
Replies
3
Views
1K
  • · Replies 2 ·
Replies
2
Views
1K
  • · Replies 3 ·
Replies
3
Views
2K
  • · Replies 8 ·
Replies
8
Views
2K
  • · Replies 1 ·
Replies
1
Views
5K
  • · Replies 7 ·
Replies
7
Views
2K
  • · Replies 25 ·
Replies
25
Views
3K